MARKING BRAINLEIST PLS HELP ASAP

MARKING BRAINLEIST PLS HELP ASAP

Answers

Answer 1

Answer:

Step-by-step explanation:

a coordinate plane has given a coordinate to each an every point we draw on that. So when we have to find the radius of this circle , we have to find a pair of coordinates.

So I am taking these coordinates; (0,3) and (0,7)

So the radius = 7-3= 4 units


Related Questions

What is the sum of the polynomials?

Answers

The sum of the given polynomial  [tex](7x^{3} - 4x^{2} ) + (2x^{3} - 4x^{2} ).[/tex]is [tex]9x^{3} - 8x^{2}[/tex].

How to add the polynomials?

Polynomials can be added or subtracted when they contain like terms, which are the same alphabets with the same powers.

Whereas there are no such rules for multiplication, any phrase with any power can be multiplied. The powers of the terms are then modified in accordance with exponentiation principles.

The following polynomial equation is [tex](7x^{3} - 4x^{2} ) + (2x^{3} - 4x^{2} ).[/tex]

Polynomial addition is accomplished by combining 'like terms'.

As a result, sorting and adding

= [tex](7x^{3} - 4x^{2} ) + (2x^{3} - 4x^{2} ).[/tex]

= [tex]7x^{3} + 2x^{3} - 4x^{2} - 4x^{2}[/tex]

= [tex]9x^{3} - 8x^{2}[/tex]

Learn more about polynomials here:

https://brainly.com/question/11536910

#SPJ1

Complete question:

What is the sum of the polynomials? (7x3 - 4x2) + (2x3 - 4x2)

Michelle was instructed to write two equivalent expressions for 6x + 15.
Her work is shown.


6x + 15 = x + x + x + x + x + x + 15


6x + 15 = 6(x + 15)


Part A: Explain which one of Michelle’s equations is true for all values of x and which one of Michelle’s equations is false for all values of x. (2 pts.)


Part B: Write another equivalent expression for 6x + 15.

Answers

Michelle’s equations that is true for all values of x is 6x + 15 = x + x + x + x + x + x + 15.

Michelle’s equations that is false for all values of x is 6x + 15 = 6(x + 15).

Another equivalent expression for 6x + 15 is 3(2x + 5).

What is an expression?

In Mathematics and Geometry, an expression simply refers to a type of mathematical equation which is typically used for illustrating the relationship that exist between two (2) or more variables and numerical quantities (number), without an equal to sign (symbol).

Michelle’s equations can be written for all values of x as follows;

6x + 15 = x + x + x + x + x + x + 15.

However, Michelle’s equations is false for all values of x when it is written as 6(x + 15).

Another equivalent expression is given by;

6x + 15 = 3(2x + 5).

Read more on expression here: https://brainly.com/question/31442748

#SPJ1

Answer:

Part A: The equation 6x + 15 = 6(x + 15) is true for all values of x because it distributes the 6 to both terms inside the parentheses and simplifies to the original expression. However, the equation 6x + 15 = x + x + x + x + x + x + 15 is false for all values of x. While it may be true for specific values of x, such as x = 1 or x = 2, it is not true for all values of x because the number of x terms changes depending on the value of x.

Part B: Another equivalent expression for 6x + 15 could be 3(2x + 5), which also simplifies to the original expression. This is because we can factor out a common factor of 3 from both terms and simplify to 3(2x + 5) = 6x + 15.

Step-by-step explanation:

The expedition has a 28 gallon tank and gets 17 mpg. It can drive miles before running out of gas

Answers

On solving the provided question we can say that equation  As a result, the expedition can go 476 kilometres before running out of fuel.

What is equation?

A mathematical equation is a formula that connects two statements and denotes equivalence with the equals symbol (=). An equation is a mathematical statement that shows the equality of two mathematical expressions in algebra. In the equation 3x + 5 = 14, for example, the equal sign separates the variables 3x + 5 and 14. A mathematical formula describes the connection between the two sentences that occur on opposite sides of a letter. The symbol and the single variable are frequently the same. As in 2x - 4 equals 2, for instance.

To calculate how far the expedition can travel before running out of gas, apply the following formula:

Miles = (Gallons of gas) x (Miles per gallon)

Plugging in the given values, we get:

Miles = 28 gallons x 17 mpg

Miles = 476 miles

As a result, the expedition can go 476 kilometres before running out of fuel.

To know more about equation visit:

https://brainly.com/question/649785

#SPJ1

Find the surface area of the composite solid.

Answers

The surface area of the figure is derived to be equal to 656.4 square meters.

How to evaluate the surface area of a cylinder

The top of the figure is a triangular prism, the bottom is a triangle, and the sides are 3 same size rectangles, so the surface area is calculated as follows:

area of one top triangle face = 1/2 × 12 m × 5 m = 30 m²

area of top triangular prism = 3 × 30 m² = 90 m².

area of bottom triangle = 1/2 × 12 m × 10.4 m = 62.4 m²

area of one rectangle side = 14 m × 12 m = 168 m²

area of the three rectangle sides = 3 × 168 m² = 504 m²

surface area of the figure = 90 m² + 62.4 m² + 504 m²

surface area of the figure = 656.4 m².

Therefore, the surface area of the figure is derived to be equal to 656.4 square meters.

Read more about surface area here:https://brainly.com/question/27440983

#SPJ1

Each input value, x, is squared and then 3 is added to
the result. The domain of the function is [0, ∞).


make a graph or equation

Answers

The equation of the function is f(x) = x² + 3, where x represents the input value and f(x) represents the output value. Graph is in below figure.

What is a domain of the function?

The domain of a function is the set of all possible input values (also called the independent variable) for which the function is defined. In other words, it is the set of values that can be plugged into the function to produce a valid output value (also called the dependent variable).

The equation of the function is f(x) = x² + 3, where x represents the input value and f(x) represents the output value.

To graph this function, we can plot points by choosing values of x and computing the corresponding values of f(x). Since the domain of the function is [0, ∞), we will only consider non-negative values of x.

x f(x)

0 3

1 4

2 7

3 12

4 19

We can plot these points on a coordinate plane and connect them with a smooth curve to obtain the graph of the function:

Note that the graph is a parabola that opens upwards and passes through the point (0, 3).

To know more about graph, visit:

https://brainly.com/question/11803331

#SPJ1

since critical values of t vary by sample size, before using the t table we must first calculate

Answers

Critical values of t, which are used for hypothesis testing and confidence interval calculations, vary depending on the sample size and the degrees of freedom.

b) degrees of freedom.

Degrees of freedom (df) are calculated based on the sample size and represent the number of independent pieces of information available in the data. It is an important parameter in determining the appropriate critical value to use from the t-distribution table. Therefore, before using the t-table, it is essential to calculate the degrees of freedom of the data set in question.

Options a, c, and d are not correct answers as they do not pertain to the calculation needed before using the t-table.

Learn more about “ Critical values “ visit here;

https://brainly.com/question/30168469

#SPJ4

Complete Question

Since critical values of t vary by sample size, before using the t table we must first calculate:

a) the Z score.

b) degrees of freedom.

c) the population standard deviation.

d) the sample size.

The degrees of freedom will then determine the critical values of t that should be used in calculating confidence intervals or conducting hypothesis tests. Therefore, it is important to consider the sample size when working with t statistics and using the t table.

To answer your question, before using the t-table, you must first calculate the degrees of freedom. The degrees of freedom are important as they determine the critical values of t based on your sample size. Here's a step-by-step explanation:

1. Obtain your sample data.
2. Determine the sample size (n) by counting the number of data points in your sample.
3. Calculate the degrees of freedom (df) by subtracting 1 from the sample size: df = n - 1.
4. Refer to the t-table with the calculated degrees of freedom to find the critical values of t for your desired level of confidence or significance.

By following these steps, you'll be able to find the appropriate critical values of t based on your sample size before using the t-table.

to learn more about the sample click here:

https://brainly.com/question/15659544

#SPJ11

The proportion of scores in a standard Normal Distribution that are greater than 1.25 is closest to...a) .1056b) .1151c) .1600d) .8849e) .8944

Answers

The proportion of scores in a standard Normal Distribution that are greater than 1.25 is closest to a. a is 0.1056.

The proportion of scores in a standard Normal Distribution that are greater than 1.25 is:

[tex]P(Z > 1.25) = 1 - P(Z < 1.25)[/tex]

Using a standard normal distribution table or a calculator.

The area to the left of 1.25 is 0.8944, so:

[tex]P(Z > 1.25) = 1 - P(Z < 1.25) = 1 - 0.8944 = 0.1056[/tex]

The closest answer choice is (a) 0.1056.

The percentage of scores in a normal distribution with a standard deviation larger than 1.25 is:

either a calculator or a normal distribution standard table.

0.8944 is the region to the left of 1.25, so:

The nearest possible response is (a) 0.1056.

For similar questions on Scores

https://brainly.com/question/28000192

#SPJ11

Given the triangle below...
30°
B
79
A
What is the measure of angle B?

Answers

Answer: 71

Step-by-step explanation: to get the answer you have to know that all the angles of a triangle have to add up to 180 so therefore you add the angles they gave you and then you minus them from 180 and you get 71

A car has 12,500 miles on its odometer. Say the car is driven an average of 40 miles per day. Choose the model that expresses the number of miles N that will be on its odometer after x days Choose the correct answer below ( A. N(x)=12.500x + 40 OB. N(x)--40x + 12,500 OC. Nx)-40-12,500 O D. N(x)-40x+12,500

Answers

The correct answer is B.

N(x) = 40x + 12,500.

This is because we know that the car is driven an average of 40 miles per day, so after x days, it will have traveled a total of 40x miles.

Adding this to the initial 12,500 miles on the odometer gives us the expression N(x) = 40x + 12,500 for the total number of miles on the car's odometer after x days.

Based on the given information, the correct answer is B.

N(x) = 40x + 12,500
This model represents the total number of miles (N) on the odometer after x days, taking into account the initial 12,500 miles and the average daily increase of 40 miles.

To know more about average: brainly.com/question/27851466

#SPJ11

The correct answer is B.N(x) = 40x + 12,500.

This is because we know that the car is driven an average of 40 miles per day, so after x days, it will have traveled a total of 40x miles.

Adding this to the initial 12,500 miles on the odometer gives us the expression N(x) = 40x + 12,500 for the total number of miles on the car's odometer after x days.

Based on the given information, the correct answer is B.

N(x) = 40x + 12,500

This model represents the total number of miles (N) on the odometer after x days, taking into account the initial 12,500 miles and the average daily increase of 40 miles.

To know more about average:

brainly.com/question/27851466

#SPJ4

smo help me fill in the charts, will give brainliest

Answers

Answer:

In the explanation

Hope this helps!

Step-by-step explanation:

a.

1. LCM of 2 & 5 is 10

2. [tex]\frac{15x}{10} =\frac{12}{10}[/tex]  -> 15x = 12

3. x = [tex]\frac{12}{15}[/tex] or [tex]\frac{4}{5}[/tex]

b.

1. LCM of 3 & 9 is 9

2. 2x - [tex]\frac{6}{9}[/tex] = [tex]\frac{4}{9}[/tex]  ->  2x = [tex]\frac{10}{9}[/tex]  ->  [tex]\frac{18}{9} x[/tex] = [tex]\frac{10}{9}[/tex]  ->  x = [tex]\frac{10}{9}[/tex] × [tex]\frac{9}{18}[/tex]  ->  x = [tex]\frac{10}{18}[/tex]  ->  x = [tex]\frac{5}{9}[/tex]

3. x = [tex]\frac{5}{9}[/tex]

In Rodger's state, unemployment compensation is calculated by finding the total of the quarterly wages of two consecutive quarters and dividing by 26. The weekly
unemployment is 65% of that amount. In the quarter including January, February, and March, Rodger made a total of $13,950.80. In the quarter including April, May, and
June, he made a total of $14,250.10. Find Rodger's weekly unemployment amount.

Answers

Rodger's weekly unemployment amount will be $ 705.0225

Given that the total of the quarterly wages of two consecutive quarters and divided by 26.

The weekly unemployment is 65% of that amount.

Rodger made a total of $13,950.80.

Then he made a total of $14,250.10

Step 1:  the total of the quarterly wages of two consecutive quarters.

= $ 13,950.80 + $ 14,250.10

= $28,200.90

Now Divide the total  by 26

 = $28,200.90 / 26

 =  1,084.65

Then,  =   1,084.65 x 65%

 =  1,084.65 x 65/100

=   705.0225

Hence, Rodger's weekly unemployment amount is; $ 705.0225

Learn more about Wages at:

brainly.com/question/13847060

#SPJ1

What is the value of x in the right triangle below?
Show your work and round your answer to the nearest hundredth.

Answers

The value of x in the right triangle below by using Pythagorean Theorem (H² = P² + B²) is 19.53

According to question; Perpendicular (P) = 16, Base (B) = 11.2 and Hypoteneus (H) = X

According to Pythagorean Theorem

H² = P² + B²

X² = 16² + 11.2²

X² = 256 + 125.44

X² = 381.44

X = 19.53

The Pythagorean Theorem is what?

The Pythagorean Theorem, or, in standard algebraic notation, a² + b² = c², states that the total of the squares on the legs of a right triangle is equal to the square on the hypotenuse (the side opposite the right angle).

To know more about Pythagorean Theorem visit:

https://brainly.com/question/14930619

#SPJ1

A suitcase measures 24 inches long and 18 inches high what is the diagonal length of the suitcase to the nearest 10th of a foot

Answers

Answer: Split it in half to turn it into a triangle! The hypotenuse if the triangle will give you the diagonal length of the suitcase, because it’s a rectangle. Hope this helps :D

Step-by-step explanation:

Where will the parabola intersect the​ line? What equation did you solve to find the​ intersection? Question content area bottom
Part 1 Use the equation to find the intersection.
The parabola will intersect the line at____enter your response here.

Answers

From the graph of the parabola, line, and the given values we found out that the parabola will intersect the line at coordinates ([tex]2\sqrt{14}[/tex],7).

What is a parabola and how do we obtain the equations of the same?A parabola is a type of curve that can be described as a symmetrical U-shape. The generalized equation of a parabola is [tex]y=ax^{2} +bx+c[/tex], where a, b, and c are constant values.This equation represents a vertical parabola with its vertex at the point [tex](-b/2a, c - b^2/4a)[/tex] and an axis of symmetry that is parallel to the y-axis.This equation represents a parabola that opens to the right or left, with its vertex at the point [tex](c - b^2/4a, -b/2a)[/tex] and an axis of symmetry that is parallel to the x-axis. The specific coefficients in the equation depend on the particular characteristics of the parabola, such as its vertex, axis of symmetry, and whether it opens up or down or left or right.

As the parabola is originating from the origin and opens at the y-axis, hence:

Equation of parabola = [tex]x^{2} =4ay[/tex] {where a =2} .....................(1)Equation of the line=[tex]y-7=0[/tex].............................................(2)From equation 2 we obtain y=7 and will put this value in 1.After computing the above values in 1 we get: [tex]x^{2} =4*2*7\\x=\sqrt{4*2*7}\\ x=2\sqrt{14}[/tex]  and y=7So the coordinates at which the parabola intersects the line is  [tex](2\sqrt{14} ,7)[/tex]

To know more about the parabola visit:

https://brainly.com/question/31142122

#SPJ1

o determine whether voters would be willing to vote for a tax increase to improve the city's parks, the park commission surveys 500 park users.

Why is this sample likely to be biased?

Answers

This sample is likely to be biased because it only includes individuals who use the city's parks, which means it may not represent the entire population of voters who would be affected by a tax increase to improve the parks.

What is sample?

A sample is a subset or a smaller group of individuals or 17 that are selected from a larger population to represent the population.

According to question:

The park commission's study of 500 park users is likely biassed because it only covers people who use the city's parks. This indicates that the sample does not accurately reflect the full electorate who would be impacted by a tax hike to fund park improvements.

Various people may hold various views regarding whether or not a tax increase is required to develop the parks. For instance, if they don't frequent the parks or have other priorities for their tax money, they might be less likely to approve a tax hike.

If the park commission only conducted surveys of people at specific times of day or in specific locations of the city, the sample may also be skewed. This could omit those who visit the parks at various hours or in various locations around the city.

In order to get accurate and valid results, it's crucial to utilise a representative sample that includes people from different demographic groups. A biased sample can produce results that are erroneous or misleading.

To know more about sample visit:

https://brainly.com/question/11045407

#SPJ1

need help ASAP!!! Please help! I’m having trouble!

Answers

The Surface Area of wooden box is 432 inch².

We have,

length = 12 inch

width = 8 inch

Height = 6 inch

So, Surface Area of wooden box

= 2 (lw + wh + lh)

= 2 (96 + 48 + 72)

= 2 x 216

= 432 inch²

Learn more about Surface Area here:

https://brainly.com/question/29298005

#SPJ1

The time $t$t​ (in seconds) it takes a dropped object to fall h$h$h​ feet is given by t is equal to square root of h over 16 end root$t=\sqrt{\frac{h}{16}}$t=√
h
16​​ .

A building of height 55 feet.
a. Estimate how long it takes an earring to hit the ground when it falls from the roof of the building. Round your answer to the nearest hundredth.

about ____ seconds

b. Estimate how much sooner the earring hits the ground when it is dropped from two stories (22 feet) below the roof. Round your answer to the nearest hundredth.

about seconds

Answers

a. About 3.01 seconds.b. The earring hits the ground about 0.69 seconds sooner when dropped from two stories below the roof.

Kyle who is a lifeguard is roping off a rectangular swimming area of 70 m. He uses the beach as one side.

Determine the minimum length, to 2 decimal places, of rope needed.

Answers

If he uses the beach as one side. the minimum length, to 2 decimal places, of rope needed is: 70 meters of rope.

What is the  minimum length?

Let's assume that the length of the swimming area is L and the width is W. The perimeter of the swimming area will be the length of the rope that Kyle needs to rope off the area.

From the problem, we know that one side of the swimming area is the beach, so we have:

L + W + L = 70

Simplifying this equation, we get:

2L + W = 70

To find the minimum length of rope needed, we need to minimize the perimeter of the swimming area. We can use the above equation to express W in terms of L:

W = 70 - 2L

Substituting this expression for W into the formula for the perimeter, we get:

P = L + W + L

= 2L + (70 - 2L)

= 70

So the perimeter of the swimming area is fixed at 70 m, regardless of the values of L and W. Therefore, the minimum length of rope needed to rope off the area is simply the perimeter of the swimming area, which is 70 meters.

Therefore, Kyle needs a minimum of 70 meters of rope.

Learn more about  minimum length here:https://brainly.com/question/5716477

#SPJ1

Solve for A and B using special right triangles

Answers

Answer:

a = 10

b = 5

Step-by-step explanation:

A 30-60-90 triangle is a right triangle where the interior angles measure 30°, 60° and 90° and the sides are in the ratio 1 : √3 : 2.

The formula for the ratio of the sides is x : x√3 : 2x where:

x = the shortest side opposite the 30° angle.x√3 = the side opposite the 60° angle.2x = the longest side (hypotenuse) opposite the right angle.

From inspection of the given triangle, the measure of the side opposite the 30° angle is "b", the side opposite the 60° angle is 5√3, and the hypotenuse is "a". Therefore:

x = bx√3 = 5√3 2x = a

If x√3 = 5√3, then x = 5. Therefore:

b = x = 5a = 2x = 10

suppose lengths of text messages are normally distributed and have a known population standard deviation of 3 characters and an unknown population mean. a random sample of 22 text messages is taken and gives a sample mean of 31 characters. what is the correct interpretation of the 90% confidence interval?

Answers

Answer: The 90% confidence interval for the population mean of the length of text messages is a range of values that is likely to contain the true population mean with a probability of 0.90 (or 90%). Based on the given information, we can calculate the confidence interval as follows:

Standard error of the mean (SE) = σ / sqrt(n)

where σ is the population standard deviation, n is the sample size, and sqrt denotes the square root.

SE = 3 / sqrt(22) ≈ 0.639

Margin of error (ME) = t(α/2, df) × SE

where t(α/2, df) is the critical value from the t-distribution with df degrees of freedom, and α is the level of significance (1 - confidence level).

For a 90% confidence level and 21 degrees of freedom (df = n - 1), the critical value is approximately 1.717.

ME = 1.717 × 0.639 ≈ 1.098

The confidence interval can be calculated as:

CI = sample mean ± ME

= 31 ± 1.098

= (29.902, 32.098)

Therefore, we can say that we are 90% confident that the true population mean of the length of text messages falls between 29.902 and 32.098 characters. In other words, if we were to repeat the sampling process many times and construct a 90% confidence interval for each sample, we would expect 90% of the intervals to contain the true population mean. Additionally, we can interpret the margin of error as the maximum amount that the sample mean is expected to differ from the true population mean, with a probability of 90%.

Step-by-step explanation:

he diameter of a semicircle is 23 centimeters. What is the semicircle's area?

Answers

Answer: in2

Step-by-step explanation:

answerd

A rectangle is 5√7+2√3 meters long and 6√7-3√3 meters wide.
a. Find the perimeter of the rectangle in simplest form.
b. Find the area of the rectanglè in simplest form

Answers

The a. Perimeter = 22√7 - 2√3 meters. The Area = 4√12 - 3√7 square meters

How to Find the Area and Perimeter of a Rectangle?

Area = length * width

Perimeter = 2(length + width)

Given the following:

Length of the rectangle = 5√7 + 2√3 meters

Width of the rectangle = 6√7 - 3√3 meters

a. Perimeter = 2(5√7 + 2√3) + 2(6√7 - 3√3)

10√7 + 4√3 + 12√7 - 6√3

22√7 - 2√3 meters

b. Area = (5√7 + 2√3)(6√7 - 3√3)

= 5√7(6√7 - 3√3) + 2√3(6√7 - 3√3)

= 210 - 15√21 + 12√21 - 18

= 192 - 3√7

= 4√12 - 3√7 square meters

Learn more about the rectangle on:

https://brainly.com/question/13048427

#SPJ1

can someone help me w this

Answers

For A in order from top to bottom the answers are: 3, 6, 24

For B in order from top to bottom the answers are:16, 80, 64, 200

Explanation:

For a you are plugging in for m them dividing

24/8 = 3
24/4 = 6
24/1 = 24

For b you are plugging in for y and multiplying

8(2) = 16
8(10) = 80
8(8) = 64
8(25) = 200

Hope this helps!

11. Tyrell buys an organizer for his baseball cards that costs $12.99. He can
add pages to the organizer to hold the cards. Each page costs $2.75 and
holds 9 cards. If Tyrell has 100 cards, how much will it cost him to organize
them? Write and evaluate a numeric expression.

Answers

On solving the provided question we can say that - 12.99(y - 9) + 2.75y = 100 , by solving, the expression  x = 11.2, y = 19

What is  linear equation?

A linear equation in algebra is one that only contains a constant and a first-order (direct) element, such as y = mx b, where m is the pitch and b is the y-intercept.

                         Sometimes the following is referred to as a "direct equation of two variables," where y and x are the variables. Direct equations are those in which all of the variables are powers of one. In one example with just one variable, layoff b = 0, where a and b are real numbers and x is the variable, is used.

let x be the base ball cards.

and y be page card.

x + y = 9 -------------------1

12.99x + 2.75y = 100-------------------2

12.99(y - 9) + 2.75y = 100

by solving

x = 11.2

y = 19

Learn more about Linear equation

brainly.com/question/11897796

#SPJ1

Simplify [tex]\frac{\sqrt 7 + \sqrt 3}{2\sqrt 3 - \sqrt 7}[/tex]

Answers

The simplified rational expression for this problem is given as follows:

[tex]\frac{3\sqrt{21} + 13}{12}[/tex]

How to simplify the rational expression?

The rational expression in the context of this problem is defined as follows:

[tex]\frac{\sqrt{7} + \sqrt{3}}{2\sqrt{3} - \sqrt{7}}[/tex]

The first step in simplifying the expression is removing the root from the denominator, multiplying numerator and denominator by the conjugate, as follows:

[tex]\frac{\sqrt{7} + \sqrt{3}}{2\sqrt{3} - \sqrt{7}} \times \frac{2\sqrt{3} + \sqrt{7}}{2\sqrt{3} + \sqrt{7}}[/tex]

Applying the subtraction of perfect squares, the denominator is given as follows:

2² x 3 - 7 = 12.

The numerator is:

[tex](\sqrt{7} + \sqrt{3})(2\sqrt{3} + \sqrt{7}) = 2\sqrt{21} + 7 + 6 + \sqrt{21} = 3\sqrt{21} + 13[/tex]

Thus the simplified expression is:

[tex]\frac{3\sqrt{21} + 13}{12}[/tex]

More can be learned about rational expressions at https://brainly.com/question/29061047

#SPJ1

Steve wants to start his own T-shirt business. His initial costs to get the business going are $1,500. It also costs on average $3 per shirt. Steve charges $10 for each shirt he sells. Which of the following systems is correct for this model to find the break-even point if "x" = the number of shirts sold and "y" = the number of dollars of expense or income?

Answers

Answer:

Below

Step-by-step explanation:

To find the break-even point, we need to determine the point at which the total revenue equals the total cost. Let's call the number of shirts sold "x".

The total cost includes the initial costs of $1,500 and the cost per shirt of $3x, so the total cost is:

y = 1500 + 3x

The total revenue is the price per shirt of $10 multiplied by the number of shirts sold, so the total revenue is:

y = 10x

To find the break-even point, we need to set the total cost equal to the total revenue and solve for x:

1500 + 3x = 10x

Simplifying this equation, we get:

1500 = 7x

Dividing both sides by 7, we get:

x = 214.29

Therefore, Steve needs to sell 215 shirts to break even.So the correct system to find the break-even point is:

y = 1500 + 3x (total cost)

y = 10x (total revenue)

Choose a Strategy. Ms. Jimenez earns $27,000 per year. She is paid weekly. She puts 8% of her salary in a retirement fund. How much money goes into this fund each week?

Answers

After answering the presented question, we may conclude that  expressions As a result, Ms. Jimenez contributes roughly $41.54 to her retirement fund each week.

what is expression ?

In mathematics, you can multiply, divide, add, or subtract. An expression is constructed as follows: Number, expression, and mathematical operator A mathematical expression (such as addition, subtraction, multiplication, or division) is made up of numbers, variables, and functions. It is possible to contrast expressions and phrases. An expression or algebraic expression is any mathematical statement that has variables, integers, and an arithmetic operation between them. For example, the phrase 4m + 5 has the terms 4m and 5, as well as the provided expression's variable m, all separated by the arithmetic sign +.

We need to do the following to figure out how much money Ms. Jimenez puts into her retirement fund each week:

Ms. Jimenez's yearly retirement fund contribution is calculated as follows:

8% of $27,000 = annual retirement fund contribution

= 0.08 x $27,000

= $2,160

To calculate the weekly retirement fund contribution, divide the yearly payment by the number of weeks in the year:

Contribution to a retirement fund on a weekly basis = Annual contribution to a retirement fund divided by the number of weeks in a year

= $2,160 / 52

≈ $41.54

As a result, Ms. Jimenez contributes roughly $41.54 to her retirement fund each week.

To know more about expressions visit :-

https://brainly.com/question/14083225

#SPJ1

Weights of the Pacific yellowfin tuna follow a normal distribution with mean weight 68 pounds and standard deviation 12 pounds. For a randomly caught Pacific yellowfin tuna, what is the probability that the weight is less than 50 pounds

Answers

The probability that the weight of a randomly caught Pacific yellowfin tuna is less than 50 pounds is approximately 0.0668 or 6.68%.



The weights of the Pacific yellow fin tuna follow a normal distribution with a mean weight of 68 pounds and a standard deviation of 12 pounds. To find the probability that a randomly caught Pacific yellow fin tuna weighs less than 50 pounds, we can follow these steps:

1. Calculate the z-score for the weight of 50 pounds using the formula:
z = (X - μ) / σ
where X is the weight, μ is the mean, and σ is the standard deviation.

2. Find the probability associated with the z-score using a z-table or calculator.

Let's calculate the z-score:
z = (50 - 68) / 12
z = -18 / 12
z = -1.5

Now, we can use a z-table or calculator to find the probability associated with the z-score of -1.5. The probability that a randomly caught Pacific yellow fin tuna weighs less than 50 pounds is approximately 0.0668, or 6.68%.

Know more about probability here:

https://brainly.com/question/13604758

#SPJ11

What is the most common type of employee benefit?

O retirement
O disability insurance
O life insurance
O healthcare

Answers

Answer:

life insurance is the most common type of employee benefit

why does an angle formed by a tngent and a chord have the same measure as an insribed angle that intercepts the same arc

Answers

An angle formed by a tangent and a chord in a circle is called an "angle between tangent and chord" while an inscribed angle is an angle whose vertex is on the circle and whose sides intersect the circle at two distinct points.

When a tangent line and a chord intersect at a point on the circle, the tangent line is perpendicular to the radius drawn to the point of intersection.

This means that the angle between the tangent and the chord is equal to the angle between the radius and the chord.

Now, consider an inscribed angle that intercepts the same arc as the chord.

By the inscribed angle theorem, the measure of an inscribed angle is half the measure of the arc that it intercepts.

Thus, the measure of the inscribed angle is equal to the measure of the arc that the chord intercepts.

Since the angle between the tangent and the chord is equal to the angle between the radius and the chord, and the inscribed angle intercepts the same arc as the chord, it follows that the angle between the tangent and the chord is equal to the inscribed angle that intercepts the same arc.

Therefore, an angle formed by a tangent and a chord has the same measure as an inscribed angle that intercepts the same arc.

For similar question on tangent.

https://brainly.com/question/28049264

#SPJ11

Other Questions
a teacher wants to improve literacy by planning a new vocabulary lesson with her class. which activity would be appropriate to incorporate into her lesson? the people who invented opera believed that they were re-creating the greek tragic style, which had fused music with drama. true or fasle You find a novel organism with unique structure and begin to study its nervous system. You find that Vm = -40 mV. Important ions in this system appear to be magnesium (Mg^2+) and Rubidium (Rb^+). Ion: Rb^+Intracellular Concentration (mM): 200Extracellular Concentration (mM): 5Ion: Mg^2+Intracellular Concentration (mM): 50Extracellular Concentration (mM): 400For this system, the concentration gradient for magnesium favors [] and the electrical gradient favors [y].A) efflux; effluxB) influx; effluxC) influx; no effectD) influx; influxE) efflux; influx what is the missing number. Good points What is the thesis statement? There is a new cause for concern for parents who think they are feeding their young children safe and healthy foods a new lawsuit claims the environmental protection agency is allowing unacceptably high levels of pesticides in some foods favored by children Massachusetts joins Connecticut New Jersey and New York and alleging everything from grapes to oranges to potatoes contains pesticide residues that are excessive for children the EPA is being accused of not setting child safe pesticide limits as required by the food quality protection act of 1996 these safety standards exist theyve existed since 1996 they need to be enforced we need to have these levels set and ideally theyll be set at 10 times the safety standards so that they will protect kids said Massachusetts assistant attorney general Alice Moore. VA student dissolves 11.S g of sodium hydroxide (NaOH) in 250. g of water in a well-insulated open cup. He then observes the temperature of the water risefrom 20.0 C to 31.3 C over the course of 6.7 minutes.Use this data, and any information you need from the ALEKS Data resource, to answer the questions below about this reaction:NaOH(s) -. Na (ag) + OH (ag)You can make any reasonable assumptions about the physical properties of the solution. Be sure answers you calculate using measured data are rounded to 3significant digits.Note for advanced students: it's possible the student did not do the experiment carefully, and the values you calculate may not be the same as the known andpublished values for this reaction.is this reaction exothermic, endothermic, or neither?OexothermicO endothermicO neither0.If you said the reaction was exothermic or endothermic, calculate the amount ofheat that was released or absorbed by the reaction in this case.Calculate the reaction enthalpy AH.nen per mole of NaOH.kJ A cafe sells small, medium and large mugs of coffee. A small mug has a capacity of 100 ml. A medium mug has double the capacity of a small mug. A large mug has double the capacity of a medium mug. Work out the capacity, in litres, of a large mug. asumme the input amalog voltage is changing between -5 to 5V;using a10bit A/D cconverter.calculate the number of quantization levels.calculate the voltage resolution I need help with this question (4x+3)5=2x/6 Discuss the expectations-augmented Phillips curve and the role it plays inmodels such as the one presented by Barro and Gordon. What do modelslike this imply for the design of central bank institutions? What influencehave they had on international central banks over the past 20 years? The molecular formula of aspirin is C9H8O4. How many aspirin molecules are present inone 500-milligram tablet?A) 2.77 molecules D) 1.67 1021 moleculesB) 2.77 10-3 molecules E) None of these is correct.C) 1.67 1024 molecules Which is a stronger acid? A) pH=4B) pH=5 int[][] arr = {{1, 3, 4}, {4, 5, 3}};int max = arr[0][0];for (int row = 0; row < arr.length; row++){for (int col = 0; col < arr[row].length; col++){int temp = arr[row][col];if (temp % 2 == 0){arr[row][col] = temp + 1; // line 11}if (temp > max){max = temp;}}}System.out.println(max);How many times will the statement in line 11 be executed as a result of executing the code segment? Kylie brought 5 pears to soccer practice to share with her teammates. She cuts each pear into thirds. How many slices of pears does she have to share with her teammates? Which equations can you use to solve the problem? Select two equations. A. 5 3 = 15 B. 1 5 1 3 = 1 15 C. 1 5 3 = 3 5 D. 5 1 3 = 15 E. 1 3 5 = 1 15 answer quick please Many travel narratives follow an arc similar to the plot of a storya conflict is introduced, intensifies, comes to a climax, and is resolved, outline the arc of the story told in this travel narrative The diagram shows Hox genes and body segments in fruit flies. Head Fruit fly embryo Thorax Hox genes Adult fruit fly Abdomen Why does the developing embryo grow wings only in the thorax segment?a. only the cells in the thorax segment contain the genes needed to build wingsb. the genes for building wings are used first in the thorax segment, and then transcription factors block them from being used againc. HOX genes produce transcription factors that regulate genes for wings, so they are turned on only in the thoraxd. after the wings develop in the thorax segment, HOX genes turn off those genes in the other segments The Honda Accord was named the best midsized car for resale value for 2018 by the Kelley Blue Book (Kelley Blue Book website). The file AutoResale contains mileage, age, and selling price for a sample of 33 Honda Accords.Price: 16998, 17599, 15998, 15998, 11599, 13998, 14998, 11599, 12599, 14998, 15998, 11599, 10499, 14599, 10599, 16998, 20998, 11599, 12599, 13599, 11599, 9599, 18998, 12599, 12599, 10998, 11998, 12998, 17599, 9998, 9998, 16998, 17599Mileage: 35000, 16000, 42000, 61000, 102000, 109000, 60000, 88000, 58000, 27000, 32000, 95000, 123000, 54000, 68000, 40000, 7000, 81000, 70000, 86000, 92000, 120000, 20000, 63000, 70000, 88000, 86000, 60000, 45000, 96000, 60000, 36000, 45000Age: 3, 3, 3, 4, 7, 4, 5, 9, 8, 6, 4, 8, 8, 5, 11, 4, 1, 9, 6, 5, 7, 10, 2, 7, 6, 10, 6, 6, 2, 11, 11, 2, 2The estimated regression equation is:Selling price = 20385.25049 - 0.03739 Mileage - 686.33668 AgeRound your answers to the nearest dollar.a. Estimate the selling price of a four-year-old Honda Accord with mileage of 40,000 miles.b. Develop a 95% confidence interval for the selling price of a car with the data in part (a).c. Develop a 95% prediction interval for the selling price of a car with the data in part (a). the cleaning-up expenses associated with pollution, which is a newly produced good. it is include in___ human reproductive behavior differs from other primates because: In this diagram, AT = 2x + 3, CT = 3x - 1, BT = x + 5,DT = 4x + 1, and mLATD = 41x + 8. If x = 2,which segment is the perpendicular bisector of the other? Explain your reasoning.